Last visit was: 24 Apr 2024, 21:39 It is currently 24 Apr 2024, 21:39

Close
GMAT Club Daily Prep
Thank you for using the timer - this advanced tool can estimate your performance and suggest more practice questions. We have subscribed you to Daily Prep Questions via email.

Customized
for You

we will pick new questions that match your level based on your Timer History

Track
Your Progress

every week, we’ll send you an estimated GMAT score based on your performance

Practice
Pays

we will pick new questions that match your level based on your Timer History
Not interested in getting valuable practice questions and articles delivered to your email? No problem, unsubscribe here.
Close
Request Expert Reply
Confirm Cancel
SORT BY:
Date
Tags:
Show Tags
Hide Tags
User avatar
Intern
Intern
Joined: 20 Sep 2020
Posts: 2
Own Kudos [?]: 1 [1]
Given Kudos: 2
Send PM
Intern
Intern
Joined: 14 Mar 2020
Posts: 6
Own Kudos [?]: 6 [1]
Given Kudos: 26
Send PM
GMAT Club Legend
GMAT Club Legend
Joined: 08 Jul 2010
Status:GMAT/GRE Tutor l Admission Consultant l On-Demand Course creator
Posts: 5957
Own Kudos [?]: 13387 [0]
Given Kudos: 124
Location: India
GMAT: QUANT+DI EXPERT
Schools: IIM (A) ISB '24
GMAT 1: 750 Q51 V41
WE:Education (Education)
Send PM
GMAT Club Legend
GMAT Club Legend
Joined: 03 Jun 2019
Posts: 5343
Own Kudos [?]: 3964 [0]
Given Kudos: 160
Location: India
GMAT 1: 690 Q50 V34
WE:Engineering (Transportation)
Send PM
Re: Is x > 10 ? (1) x - 5 > 10 (2) x +5 > 10 [#permalink]
Hi NidhiK5
Asked: Is x>10?
1) x-5>10 --> x>15
Take any number x>15 and it will satisfy x>10
SUFFICIENT

2) x+5>10 --> x>5
Take x = 6; x<10
Take x = 17; x>10
NOT SUFFICINET

IMO A

Statement 2 is NOT SUFFICIENT since it gives different results and x>10 is not always satisfied.

Hope it helps!

NidhiK5 wrote:
Is x>10?
1) x-5>10 --> x>15
2) x+5>10 --> x>5

Why is the answer statement 1 alone sufficient and not both statements together sufficient?
Both statements together is x>15 and x>5 --> upon overlapping the sets it gives x>15?
Intern
Intern
Joined: 13 Jun 2019
Posts: 14
Own Kudos [?]: 7 [0]
Given Kudos: 25
Location: Viet Nam
GMAT 1: 680 Q49 V34
Send PM
Re: Is x > 10 ? (1) x - 5 > 10 (2) x +5 > 10 [#permalink]
A is the answer

(1) x - 5 > 10
hence x>15 ---> sufficient

(2) x +5 > 10
then x>5 ---> insufficient since x can be greater than 10 or smaller than 10
Math Revolution GMAT Instructor
Joined: 16 Aug 2015
Posts: 10161
Own Kudos [?]: 16594 [1]
Given Kudos: 4
GMAT 1: 760 Q51 V42
GPA: 3.82
Send PM
Re: Is x > 10 ? (1) x - 5 > 10 (2) x +5 > 10 [#permalink]
1
Kudos
Expert Reply
Forget the conventional way to solve DS questions.

We will solve this DS question using the variable approach.

DS question with 1 variable: Let the original condition in a DS question contain 1 variable. Now, 1 variable would generally require 1 equation for us to be able to solve for the value of the variable.

We know that each condition would usually give us an equation, and Since we need 1 equation to match the numbers of variables and equations in the original condition, the logical answer is D.

Since it a key question [INTEGER QUESTION]you know the Variable Approach, you can sit back and relax when you go to take an exam.

The answer could be A, B, or D, but the default answer is D.

To master the Variable Approach, visit https://www.mathrevolution.com and check our lessons and proven techniques to score high in DS questions.

Let’s apply the 3 steps suggested previously. [Watch lessons on our website to master these 3 steps]

Step 1 of the Variable Approach: Modifying and rechecking the original condition and the question. We have to find whether x > 10.

Second and the third step of Variable Approach: From the original condition, we have 1 variable (x). To match the number of variables with the number of equations, we need 1 equation. Since conditions (1) and (2) will provide 1 equation each, D would most likely be the answer.

Let’s take a look at each condition.

Condition(1) tells us that the x - 5 > 10.

=> x - 5 > 10

=> x > 15

So, the range if x will be greater than 15 and 10 is less than 15.

=> Is x greater than 10 - YES

=>Since the answer is unique YES, condition(1)is sufficient by CMT 1.


Condition(2) tells us that the x + 5 > 10.

=> x + 5 > 10

=> x > 5

=> If x = 7 or 8 or 9 or 10 - Is x greater than 10 - NO

=> If x = 11 or 12 .. - Is x greater than 10 - YES


Since the answer is not unique YES or NO, condition(2) is not sufficient by CMT 1.


Condition(1) alone is sufficient.

So, A is the correct answer.

Answer: A
Intern
Intern
Joined: 13 Aug 2020
Posts: 25
Own Kudos [?]: 48 [3]
Given Kudos: 12
Send PM
Is x > 10 ? (1) x - 5 > 10 (2) x +5 > 10 [#permalink]
3
Kudos
x - 5 > 10, hence x>15
Which means 1st statement is sufficient

x +5 > 10, hence x>5
Which is insufficient since x can be greater than 5, i.e 6,7,8....

Hence option [A] is the answer
User avatar
Non-Human User
Joined: 09 Sep 2013
Posts: 32658
Own Kudos [?]: 821 [0]
Given Kudos: 0
Send PM
Re: Is x > 10 ? (1) x - 5 > 10 (2) x +5 > 10 [#permalink]
Hello from the GMAT Club BumpBot!

Thanks to another GMAT Club member, I have just discovered this valuable topic, yet it had no discussion for over a year. I am now bumping it up - doing my job. I think you may find it valuable (esp those replies with Kudos).

Want to see all other topics I dig out? Follow me (click follow button on profile). You will receive a summary of all topics I bump in your profile area as well as via email.
GMAT Club Bot
Re: Is x > 10 ? (1) x - 5 > 10 (2) x +5 > 10 [#permalink]
Moderator:
Math Expert
92900 posts

Powered by phpBB © phpBB Group | Emoji artwork provided by EmojiOne